Answer to Question #286993 in Calculus for Jishah

Question #286993

graphing p(x)=x⁶+2x⁵-2x³-x²

1
Expert's answer
2022-01-13T07:44:27-0500
"p(x)=x^6+2x^5-2x^3-x^2"

DomainL: "(-\\infin, \\infin)"

As "x\\to-\\infin,y\\to\\infin"

As "x\\to\\infin,y\\to\\infin"

There are no asymptotes


"p(-x)=(-x)^6+2(-x)^5-2(-x)^3-(-x)^2"

"=x^6-2x^5+2x^3-x^2"

The function "p(x)" is neither even nor odd.

"y" -intercept: "x=0, y=p(0)=(0)^6+2(0)^5-2(0)^3-(0)^2=0"

"(0,0)"


"x" -intercept(s): "y=0=>x^6-2x^5+2x^3-x^2=0"



"x^2(x^4-2x^3+2x-1)=0"

"x^2(x+1)^3(x-1)=0"

"x_1= x_2=x_3=-1, x_4=x_5=0,x_6=1"

"(-1,0), (0,0), (1,0)"

The graph passes through the origin.

Find the first derivative


"p'(x)=6x^5+10x^4-6x^2-2x"

Find the critical number(s)


"p'(x)=0=>6x^5+10x^4-6x^2-2x=0"

"2x(3x^4+5x^3-3x-1)=0"

"2x(x+1)^2(3x^2-x-1)=0"

"x_1=x_2=-1, x_3=0,"

"x_4=\\dfrac{1-\\sqrt{13}}{6}, x_5=\\dfrac{1+\\sqrt{13}}{6}"

Critical numbers: "-1,\\dfrac{1-\\sqrt{13}}{6}, 0,\\dfrac{1+\\sqrt{13}}{6}"

If "x<-1, p'(x)<0, p(x)" decreases.

If "-1<x<\\dfrac{1-\\sqrt{13}}{6}, p'(x)<0, p(x)" decreases.

If "\\dfrac{1-\\sqrt{13}}{6}<x<0, p'(x)>0, p(x)" increases.

If "0<x<\\dfrac{1+\\sqrt{13}}{6}, p'(x)<0, p(x)" decreases.

If "x>\\dfrac{1+\\sqrt{13}}{6}, p'(x)>0, p(x)" increases.


"p(-1)=(-1)^6+2(-1)^5-2(-1)^3-(-1)^2=0"

"p(\\dfrac{1-\\sqrt{13}}{6})=(\\dfrac{1-\\sqrt{13}}{6})^6+2(\\dfrac{1-\\sqrt{13}}{6})^5"

"-2(\\dfrac{1-\\sqrt{13}}{6})^3-(\\dfrac{1-\\sqrt{13}}{6})^2"

"=-\\dfrac{587-143\\sqrt{13}}{1458}"

"p(\\dfrac{1+\\sqrt{13}}{6})=(\\dfrac{1+\\sqrt{13}}{6})^6+2(\\dfrac{1+\\sqrt{13}}{6})^5"

"-2(\\dfrac{1+\\sqrt{13}}{6})^3-(\\dfrac{1+\\sqrt{13}}{6})^2="




"=-\\dfrac{587+143\\sqrt{13}}{1458}"




"p(0)=(0)^6+2(0)^5-2(0)^3-(0)^2=0"

The function "p(x)" has a local maximum with value of at "x=0."

The function "p(x)" has a local minimum with value of "-\\dfrac{587-143\\sqrt{13}}{1458}" at "x=\\dfrac{1-\\sqrt{13}}{6}."

The function "p(x)" has a local minimum with value of "-\\dfrac{587+143\\sqrt{13}}{1458}" at "x=\\dfrac{1+\\sqrt{13}}{6}."

Find the second derivative


"p''(x)=30x^4+40x^3-12x-2"

Find the point(s) of inflection.


"p''(x)=0=>30x^4+40x^3-12x-2=0"

"2(x+1)(15x^3+5x^2-5x-1)=0"

"x_1=-1, x_2\\approx-0.6795, x_3\\approx-0.1848,"

"x_4\\approx0.5310"

The function "p(x)" has the inflection points at "x=-1, x=-0.6795,"

"x=-0.1848,0.5310."

If "x<-1, p''(x)>0, p(x)" is concave up.

If "-1<x<-0.6795, p''(x)<0, p(x)" is concave down.

If "-0.6795<x<-0.1848, p''(x)>0, p(x)" is concave up.

If "-0.1848<x<0.5310, p''(x)<0, p(x)" is concave down.

If "x>0.5310, p''(x)>0, p(x)" is concave up.


Graph of the function


Need a fast expert's response?

Submit order

and get a quick answer at the best price

for any assignment or question with DETAILED EXPLANATIONS!

Comments

No comments. Be the first!

Leave a comment

LATEST TUTORIALS
New on Blog
APPROVED BY CLIENTS